What is the equation of the line that passes through the point (-1,4)and has a slope of -5?

Answers

Answer 1

Answer:

y = -5x - 1

Step-by-step explanation:

NOTE: This equation is in slope-intercept form. The equation in point-slope form is y - 4= -5(x + 1).

The formula for calculating slope-intercept form is y = mx + b.

Substitute the slope and the x and y intercepts for the equation: 4 = -5(-1) + b

4 = 5 + b

Subtract 5 from both sides.

So, -1 = b.

Now substitute the slope for m and -1 for b.

y = -5x + -1

= y = -5x - 1


Related Questions

Ashish deposite rs 1000 every month is a recurring deposit account for period of 12 months. If the bank pays interest at a certain rate p.A. And ashish gets 12715 as the maturity value of this account at what rate of interest did he pay every month

Answers

Solution :

Given :

Principal amount, P = Rs. 1000

Time period = 12 months

The maturity value = Rs. 12,715

We know that,

[tex]$ SI = \frac{PTR}{100}$[/tex]

[tex]$SI = 1000 \times \frac{n(n+1)}{2 \times 12} \times \frac{R}{100}$[/tex]

[tex]$SI = 1000 \times \frac{12(12+1)}{2 \times 12} \times \frac{R}{100}$[/tex]

SI = 65 R

So we know,

maturity value = principal amount + SI

12715 = 1000 + 65 R

65 R = 12715 - 1000

65 R = 11715

R = 18%        

So the rate is 18%

                           

Answer:

[tex]R=11\%[/tex] p.a.

Step-by-step explanation:

Given:

the principal amount deposited each month, [tex]P=Rs. 1000[/tex]

amount after maturity of one year, [tex]A=Rs. 12715[/tex]

We have the formula as:

[tex]I=\frac{PR}{100}\times\frac{T(T+1)}{2\times 12}[/tex]

where:

R = rate of interest per annum

T = time in months

[tex]A-12P=\frac{PR}{100}\times\frac{T(T+1)}{2\times 12}[/tex]    [since the principal is deposited each month]

[tex]715=\frac{1000\times R}{100}\times \frac{12\times 13}{24}[/tex]

[tex]R=11\%[/tex] p.a.

the hypotenuse of a right angled triangular field is 50ft and the legs are in the ratio 7:24, find the area of the right angled triangular field triangular field also find the cost of paving the field with brick at the rate of rs.20per square ft

Answers

Answer:

Step-by-step explanation:

If one leg is 7x than other leg is 24x

Using Pythagoras

50² = (7x)² + (24x)²

2500 = 49x² + 576x²

2500 = 625x²

x² = 2500/625 = 4

x = +2 or -2 ; x is positive

Means 7 x = 14 and 24x = 48

The two sides are 14cm and 48cm

Test: 14² + 48² = 196 + 2304 = 2500= 50²

for each relation, decide whether or not it is a function

Answers

Answer:

Relation 1,2,and 4 are functions, but relation 3 not is a function.

Step-by-step explanation:

function 1 input, no function with the same input like m in relation 3.

Explain why y +1 = 1.2(x + 2) and y- 5 = 1.2(x – 3) represent the same line, despite having
different equations.

Answers

Answer:

They have the same slope

Step-by-step explanation:

The standard equation of a line in point-slope form is expressed as;

y-y0 = m(x-x0)

We can see that both equations given are written in this form with a slope of 1.2. For two lines to be equal, they must have the same slope no matter the point on the lines. Hence the two equations are equal since they have different slopes.

I having a hard time figuring this out.

Answers

Answer:

33

Step-by-step explanation:

Start by adding the 3 to the other side (2/3x=22)

Then, you divide 2/3 to cancel it out. To do that, multiply 22 by 3/2 which equals to 33.

Answer:

2/3x - 3 = 19

We first isolate the variable and solve for x, (we get rid of any constants first in the process)

2/3x - 3 = 19 the three is the constant so we get rid of it by adding three to both sides.

2/3x - 3 = 19

+3            +3

2/3x = 22

Now we divide by 2/3 on both sides

X = 33

a bag contains 3 red marbles, 5 yellow marbles, and 2 green marbles. what is the probability that you will select 2 green marbles in a row if you do not replace the first marble? A) 0.020 B) 0.02 C) 0.040 D) 0.200​

Answers

Answer:

B

Step-by-step explanation:

find the probability of picking the first green marble, which would be 2/10

then afterwards once the marble is removed, it would be 1/9

Multiply both 2/10 and 1/9 to get 0.022, which rounds to 0.02

Answer:

0.02

Step-by-step explanation:

3+5+2 = 10

to choose 1 green the probability is 2/10 = 1/5

if we do not replace it, then there are now 9 marbles left and only 1 green left, so 1/9

to find them both in a row, multiply

1/5 * 1/9 = 1/45 = 0.02222

Which is the angle of elevation from C to B?

Answers

Answer:

Angle of elevation = ∠4

Step-by-step explanation:

Angle of elevation of a point point from another point is the angle formed between the line joining these points and the horizontal line.

Therefore, angle of elevation of point B from a point C = ∠4

Option with angle 4 will be the answer.

help help............​

Answers

Answer:

please send the pic again clearly

Answer:

I hope it will help you

Step-by-step explanation:

please make me brainlest

thank u


Which number is rational?
√2
Pi
Square root of 10
Square root of 16

Answers

Answer:

[tex]\sqrt{16}[/tex] is rational number

Step-by-step explanation:

Rational number is of the form p/q, where q ≠ 0. It may be a terminating number or non terminating repeating number.

√2 is irrational number as it is non terminating non repeating number

π is irrational number as it is non terminating non repeating number

√10 is irrational number as it is non terminating non repeating number.

[tex]\sqrt{16}=\sqrt{4*4}=4[/tex]

[tex]\sqrt{16}[/tex] is a rational number

the length of a photograph is 11.4 inches if the photo is enlarged so that its length is increased by 2.25 inches what the new length​

Answers

Length=11.4inIncreased length=2.25in

We know

[tex] \\ \sf \longmapsto \: new \: length = length + increased \: length \\ \\ \sf \longmapsto \: new \: length = 11.4 + 2.25 \\ \\ \sf \longmapsto \: new \: length = 13.65in[/tex]

Two linear equations are shown.

A coordinate grid with 2 lines. The first line is labeled y equals StartFraction one-third EndFraction x plus 2 and passes through (negative 6, 0) and (0, 2). The second line is labeled y equals StartFraction 4 over 3 EndFraction minus 5.
What is the solution to the system of equations?

(7, 4)
(7, StartFraction 13 over 3 EndFraction)
(8, StartFraction 14 over 3 EndFraction)
(9, 7)

Answers

Answer:

(7, 13/3)

Step-by-step explanation:

Given the expressions

y = 1/3x + 2 and the second line y = 4/3x - 5

Equating both expressions

1/3x + 2 = 4/3x - 5

1/3x - 4/3x = -5 - 2

-3/3x = -7

-x = -7

x = 7

Substitute x = 7 into any of the equations

Using y = 1/3 x + 2

y = 1/3(7) + 2

y = 7/3 + 2

y = (7+6)/3

y = 13/3

Hence the solution to the system of the equation is (7, 13/3)

Answer:

(7,13/3) is your answer, otherwise known as answer choice B.

Step-by-step explanation:

PLEASE HELP ASAP 30 POINTS

Answers

Answer:

I don't know how to do please let me I will try solve the question

Mọi người giúp em với

Answers

Answer:

bka bla bla bla sorry I newbie

sin pi/3 __ __ pi/6 = 1/2(sin pi/2 + sin pi/6)

I think I’m just supposed to fill in the blank? (question off of a p e x) please give explanation!

Answers

Notice that

π/2 = π/3 + π/6

π/6 = π/3 - π/6

Recall the angle sum identities for sine:

sin(x + y) = sin(x) cos(y) + cos(x) sin(y)

sin(x - y) = sin(x) cos(y) - cos(x) sin(y)

By adding these together, we get

sin(x + y) + sin(x - y) = 2 sin(x) cos(y)

==>   sin(x) cos(y) = 1/2 (sin(x + y) + sin(x - y))

Now take x = π/3 and y = π/6 :

sin(π/3) cos(π/6) = 1/2 (sin(π/2) + sin(π/6))

So the blank should be filled with cos.

I only need the answer

Answers

Answer:

1

Step-by-step explanation:

The given equation of the function is y = -a·(x - h)² + 1

The positive constants of the equation = a, and h

The points the function crosses the x-axis = 2, and 4

Where the function crosses the x-axis, y = 0, and x = 2, and 4, therefore, when x = 2, we have;

y = 0 = -a·(2 - h)² + 1

When x = 4, we have;

0 = -a·(4 - h)² + 1

-a·(2 - h)² + 1 = -a·(4 - h)² + 1

-a·(2 - h)² = -a·(4 - h)²

(2 - h)² = (4 - h)²

±(2 - h) = +#±(4 - h)

When

(2 - h) is negative, and (4 - h) is positive, but the same magnitude, we have';

-(2 - h) = +(4 - h)

2·h = 4 + 2 = 6

h = 3

0 = -a·(4 - h)² + 1 = -a·(4 - 3)² + 1 = -a + 1

Therefore, a = 1

Scarlett made a profit of $250.00 with her mobile car wash company

Answers

Not enough information to solve..... Please make your question more clear

Which system of linear inequalities is graphed?

Answers

Answer:

The first one.

Step-by-step explanation:

Graph lines as if the inequalities were equal signs.

X = -3 is a vertical line at x = -3, because it's less than we shade to the left. All numbers less than -3 are to the left. The line is dashed because there is no equal to. Only less than. The line is not included in the solution set.

y = -x - 1 is a line with a y-intercept of -1 and a slope of -1. All values that are less that y are below the line. Because it's less than or equal to the line is solid.

Answer:

A

Step-by-step explanation:

The vertical line  is dotted at -3 and shaded to the left

x < -3

This gives us two choices left

A and C

The other line has a y intercept at -1 and is solid and shaded to the left

It is of the form

y ≤ mx+1  

We know the slope is negative since is goes down from left to right

The only Choice is A

Can someone help me with this math homework please!

Answers

Answer:

(-3,-6) and (-3,2)

Step-by-step explanation:

A line with an undefined slope is vertical. In this case it must have x coordinates equal to -3

Doanh nghiệp bán trả góp 1 bất động sản có giá thanh toán là 32.000trđ . Thu điều cả vốn lẫn lãi trong 10 năm với lãi suất trả chậm là 10% / năm . Xác định số vốn phải thu ở năm thứ 6 ?

Answers

Answer:

Step-by-step explanation:

If x=3 ,y=4 than what is the value?

Answers

I am pretty sure it is 91

Find the value of x that will make L||M.
6x + 8
4x + 2
X =[?]

Answers

Answer:

6x+8+4x+2=180

so!! 10x+10=180

10x=170

x=17

Answer:

[tex]x=17[/tex]

Step-by-step explanation:

The two angles labelled [tex]6x+8[/tex] and [tex]4x+2[/tex] are co-interior angles. When two parallel lines are cut by a traversal, co-interior angles are supplementary, meaning they add up to 180 degrees. Therefore, if line L is parallel to line M, [tex]6x+8[/tex] and [tex]4x+2[/tex] must be supplementary:

[tex]6x+8+4x+2=180[/tex]

Combine like terms:

[tex]10x+10=180[/tex]

Subtract 10 from both sides:

[tex]10x=170[/tex]

Divide both sides by 10:

[tex]x=\frac{170}{10}=\boxed{17}[/tex]

I'm not sure how to answer this!!

Answers

Answer:

I think it's a

Step-by-step explanation:

hsjshbwibs hshhsvsu hshsbsbus

Good Afternoon. I hope you’re well:) I need to understand how to go about solving this without a calculator, please. Thank you so much and have a blessed day!

Answers

Answer:

Step-by-step explanation:

Being able to do this requires that you understand what is being asked and that you understand domain. This is a piecewise function, made up of 4 different parts, and each part has a domain different from every other part. Now look at the solutions. We are given coordinates, x and y. So this is how this problem is done.

In a. the coordinate is (-1, 1), right? x = -1 here, so the equation that we "pick" (I'll explain that in a sec) has to have a domain where -1 will fall. The first part of this piecewise has a domain of less than or equal to -11. Is -1 less than or equal to -11? No, it is not. -1 is greater than -11, so the equation we "pick" to use will not be this one. Look at the next piece of the function and note its domain. This domain is that x is greater than -11 and less than 5. Does -1 fit in that domain? Is -1 included in that spread of numbers? Yes it is, so that is the equation we will use. By use I mean that we will plug in -1 for x and see if y = 1 (that number comes from the coordinate (-1, 1) where y = 1). The equation is y = x + 2 and plugging in -1 for x:

y = -1 + 2 so

y = 1 and this point is on the piecewise. Let's do one more example so you can see how it looks when it DOESN'T work out, ok?

Look at b. The coordinate is (-2, -10). x = -2, so the same domain, same equation: x + 2. We plug in -2 for x to find y:

y = -2 + 2 so

y = 0.  0 does not equal -10, so this point is not on that graph.

The key here is picking the equation whose domain includes your x value and evaluating the equation at that value of x to find y.

Please help me! I cant figure this out

Answers

AB is the same as A multiplied by B. In order to get this value, we need to figure out the value of A and B.

We have two equations here:

a + 5b = 2b + 13

a = 3b - 5

The second equation already has A isolated. So, we can take that equation and plug in the value for A. Doing so will allow us to solve for B.

(3b - 5) + 5b = 2b + 13

8b - 5 = 2b + 13

6b - 5 = 13

6b = 18

b = 3

Now that we know B, we can use our equation that isolated A.

a = 3(3) - 5

a = 9 - 5

a = 4

Therefore, a = 4 and b = 3, and ab = 12.

Hope this helps!

1. Take a look at the steps below for solving the
equation 4(3 - 2x) = -20.
Step 1: 4(3 - 2x) = -20
Step 2: 12 - 8x = -20
Step 3: 12-8x - 12 = -20 - 12
Step 4: - 8x + 0 = -32
Step 5:
- 8x = -32
1
Step 6. - 8x
-1
8
8
Step 7:
x = 4
1
= -32
Which property justifies the work between ...
A. Step 1 and Step 2?
B. Step 2 and Step 3?
C. Step 3 and Step 4?
D. Step 4 and Step 5?
E. Step 5 and Step 6?
F. Step 6 and Step 7?

Answers

Answer:

x= 1

Step-by-step explanation:

Work is shown in the picture

Helpppp
Graph the quadratic function y 9x Ox 140. What are the solutions of the quadratic equation 0 - 0 - 6x
140?

Answers

Answer:

                  Answer:bf                  

Step-by-step explanation:

Please help! The answer isn’t 36!
Find the value of the expression:

Answers

Answer:

3.6

Step-by-step explanation:

Hi there!

We are given this expression:

.6√36 (.6*√36)

And we want to find the value of it

.6 can be re-written as 0.6

In that case,

0.6*√36

First, simplify what's under the radical: √36, which is equal to 6 (6*6=36)

The expression then becomes:

0.6*6

Multiply those numbers together

0.6*6=3.6

Hope this helps!

Work out the mean for the data set below:
603,607

Answers

(603+607)/2 = 1210/2 = 605

X + 1 by X is polymer or not, polymer means ,write its degrees?​

Answers

X^1 is the answer .........

help on this question please! also if you can kindly explain it would be very much appreciated!!

Answers

Answer:

D. 50°

Step-by-step explanation:

We know that there are 3 line segments that are forming the angles. Based on that, we can safely say that each line segment is 180°. Given that, we can find the other 2 interior angles of the triangle. We know that one of the exterior angles is 90° so that interior angle also has to be 90° because both angles have to sum to 180. The 2nd exterior angle that we know is 140, so 180-140 = 40°. Since we know the other 2 angles of the triangle now and we know that all the angles of a triangle must sum to 180°, we add 90 and 40 to get 130 and then subtract that from 180 to get 50°. That would look something like this: 180-(90+40) = 180 - 130 = 50°. Hope this helps!

Answer

The measure of p is 50 degrees.

Explanation

You can see that p is located in a triangle.

The interior angles of a triangle always add up to 180 degrees.

So, to solve this problem, you can find the other interior angles, then subtract those from 180 to find p.

The exterior angles are 140 and 90 degrees. To find the measure of the interior angles, you subtract them from 180, (because the exterior angles and interior angles add up to 180).

The measure of the interior angles are 40 and 90 degrees; 180-140=40 and 180-90=90.

Now you can subtract the measure of the two interior angles from 180 to find p; 180-40-90=50.

Other Questions
help pls :( I only need one question to pass explain this statement "I can set limits and still be a loving person" helphelphelphelphelphelphelp (2i+1)/(1+i) is equal to 1: I hope the ............can repair our car quickly. A. mechanic B. reporter C. surgeon D. engineer 2.He got drunk after drinking .A. a school of wine B. a sip of wine C. a flock of wine D. a band of wine A new stable element with an atomic number of 120 and an atomic mass of 246 is created in a particle accelerator. Enough of this element is created to determine that it combines with chlorine in a 1:2 ratio. Element 120 should be placed in the same group as _____ on the periodic table. Find an equation of the line through the given pair of points. (-7,-5) and (-1,-9) The equation of the line is (Simplify your answer. Type an equation using x and y as the variables. Use integers or fractions for any numbers in the equation.) please help Simplify the expressions by combining like terms.30) 4x + 3-x = A physical trainer decides to collect data to see if people are actually weight changing weight during the shelter in place. He believes there will not be a meaningful change in weight due to the shelter in place order. He randomly chooses a sample of 30 of his clients. From each client, he records their weight before the shelter in place order, and again 10 days after the order. A summary of the data is below.The trainer claims, "on average, there is no difference in my clients' weights before and after the shelter in place order." Select the pair of hypotheses that are appropriate for testing this claim.H0: d = 0H1: d < 0 (claim)H0: d = 0 (claim)H1: d 0H0: d 0 (claim)H1: d = 0H0: d = 0 (claim)H1: d > 0H0: d = 0H1: d > 0 (claim)H0: d = 0H1: d 0 (claim)H0: d = 0 (claim)H1: d < 0H0: d 0H1: d = 0 (claim)b) Select the choice that best describes the nature and direction of a hypothesis test for this claim.This is a right-tail t-test for d.This is a right-tail z-test for d.This is a two-tail t-test for d.This is a two-tail z-test for d.This is a left-tail t-test for d.This is a left-tail z-test for d.c) Find the standardized test statistic for this hypothesis test. Round your answer to 2 decimal places.d) Find the P-value for this hypothesis test. Round your answer to 4 decimal places.e) Using your previous calculations, select the correct decision for this hypothesis test.Fail to reject the alternative hypothesis.Reject the alternative hypothesis.Fail to reject the claim.Reject the claim.Fail to reject the null hypothesis.Reject the null hypothesis.f) Consider the following statements related to the trainer's claim. Interpret your decision in the context of the problem (ignoring the claim) and interpret them in the context of the claim. classroom layout1.What other additional classroom furniture such as filling cabinets, bookshelves, working tables will there be?2.Do you have other specific needs that is important for your specific subject?3.How many classroom bulletin boards will you have?4.What other classroom display ideas as swimming pool around in your head? 1. In what ways was the rise of classic rock driven by technology?2. How did the teen idol phenomenon of the 1990s mirror that of the 1960s, and how did it differ? Compare the functions shown below:Which function has the greatest maximum y-value? which algebraic expression represents this word description the quotient of six and the sum of a number and eight If you were asked to measure the success of a campaign to fight for human rights, what criteria would you use? True or false: You can create a network with two computers.AnswerTrue carly walks 30 feet in seven seconds. At this rate, how many minutes will it take for carly to walk a mile if there are 5,280 feet in one mile? PLEASE HELP, WOULD BE GREATLY APPRECIATED Cu 9: 100 g dung dch MgCl2 9.5 % tc dng vi dung dch KOH 5,6 % .c dung dch A v cht rn B. Lc rn B v nung n khi lng khng i c cht C. a) Vit pt .Tnh khi lng rn C b) Tnh nng phn trm ca dung dch A. what do all organism meed? selecet three optionsnerves systema place to livebloodwater food Question 1 Matching Worth 4 points)(0703LC)Read the sentences and match each sentence the correct word or words that complete the sentence or that answer the question.MatchTermDefinitionWhere do firefighters work?A) La estacin de bomberosWhere do you take a plane?B) El vecindarioWhere do you find a group of houses?C) La gasolineraWhere do you take your car to fill up before a long trip?D) El aeropuertoQuestion 2